LSAT and Law School Admissions Forum

Get expert LSAT preparation and law school admissions advice from PowerScore Test Preparation.

 Luke Haqq
PowerScore Staff
  • PowerScore Staff
  • Posts: 747
  • Joined: Apr 26, 2012
|
#99809
Hi ashpine17!

Answer choice (D) states that C's criticism of W's argument is that "It disallows in principle any evidence that would disconfirm its conclusion." A circular argument can do this. Effectively, this is saying that W will interpret all evidence to support W's position and won't consider the possibility of contrary evidence. We don't know that W is doing this. Rather, W seems to be reaching an incorrect conclusion based on the premises.

Namely, W thinks that the failure of researchers to find a specific gene that causes manic-depression establishes that people are not genetically predisposed to manic-depression. C points out that a single gene isn't the only possible cause, and rather that the interaction of several genes might produce a genetic predisposition to manic-depression. This is why (A) accurately describes C's criticism of W's argument: "It presupposes only one possibility where more than one exists."
User avatar
 teddykim100
  • Posts: 46
  • Joined: Jan 10, 2022
|
#104327
Hello,

picking up off the above - is D wrong simply because W doesn't disallow any EVIDENCE, but rather disallows all conclusions aka CLAIMS?
User avatar
 srusty
PowerScore Staff
  • PowerScore Staff
  • Posts: 32
  • Joined: Nov 30, 2023
|
#104350
teddykim100 wrote: Wed Dec 06, 2023 4:39 pm Hello,

picking up off the above - is D wrong simply because W doesn't disallow any EVIDENCE, but rather disallows all conclusions aka CLAIMS?
Hi Teddy,

Answer Choice (D), when boiled down to simpler language, says that Chang is taking issue with the evidence that is being used by Wirth, because Wirth is disallowing evidence that would go against their conclusion. However, the stimulus has Chang clearly state that "I do not dispute your evidence."

So, Chang's criticism of Wirth's argument is not about the evidence, because Chang says so themselves. For that reason, answer choice (D) is wrong.

Hope that helps!

Get the most out of your LSAT Prep Plus subscription.

Analyze and track your performance with our Testing and Analytics Package.